Proof of Smoothness: Analytical Steps & Examples

  • Thread starter Thread starter nughret
  • Start date Start date
  • Tags Tags
    Functions Smooth
Click For Summary
To prove that the function G(x) is smooth, one must demonstrate that it is continuously differentiable at x = 0. The discussion highlights that the proof involves using properties of smooth functions, such as the fact that sums, products, and compositions of smooth functions remain smooth. Induction on the derivatives is suggested as a possible method for establishing smoothness. The specific challenge lies in proving the smoothness at the point x = 0, as the function behaves differently there compared to other values of x. Overall, the key focus is on ensuring that G(x) meets the criteria for smoothness across its entire domain.
nughret
Messages
45
Reaction score
0
My lecturer gave me a question that included giving a proof that a particular function is smooth. I have taken a course on analysis and have no problems when it comes to proof of continuity; i was just wondering what the usual steps are in proving that a function is smooth.
I would guess that it would involve some sort of induction on the derivatives but if someone could sketch out a general proof that would be grand.
 
Physics news on Phys.org
There are some rules like: sums, products and compositions of smooth functions are smooth. Usually one uses these to show that a function is smooth. Otherwise, induction on the derivatives might work.

Since it is only a small portion of your entire exercise, could you post the function?
 
Yes well this specific question is stated as such :
f(x) is a smooth function, prove the function

G(x) = f'(0) , x = 0
(f(x) - f(0))/x , otherwise

is smooth.
I previously assumed by product rule it is true that G(x) is smooth when x is not equal to zero but obviously the whole point of the question is about x=0.
 
Question: A clock's minute hand has length 4 and its hour hand has length 3. What is the distance between the tips at the moment when it is increasing most rapidly?(Putnam Exam Question) Answer: Making assumption that both the hands moves at constant angular velocities, the answer is ## \sqrt{7} .## But don't you think this assumption is somewhat doubtful and wrong?

Similar threads

  • · Replies 5 ·
Replies
5
Views
2K
  • · Replies 11 ·
Replies
11
Views
4K
  • · Replies 2 ·
Replies
2
Views
2K
Replies
7
Views
2K
Replies
14
Views
2K
  • · Replies 2 ·
Replies
2
Views
1K
  • · Replies 7 ·
Replies
7
Views
3K
  • · Replies 1 ·
Replies
1
Views
2K
  • · Replies 20 ·
Replies
20
Views
3K
  • · Replies 9 ·
Replies
9
Views
3K